série alternée coriace

Bonjour,

comme à mon habitude, quand un problème sur ilemaths m'intéresse mais résiste aux efforts de ses meilleurs membres, je viens en faire part ici : topic (mais je pourrai relayer un peu moins vite parfois histoire de laisser le temps de cogiter, je reconnais...)

Quelle est donc la nature de $\displaystyle \sum_n \dfrac{(-1)^n}{\ln(n)+\cos(n)}$ ? Les derniers posts montrent que c'est la même que celle de la série $\sum_n \dfrac{ \sin(2n+\frac12)}{(\ln(2n)+\cos(2n))(\ln(2n+1)+\cos(2n+1))}$ et après...

Bonne fin de week-end !
«1

Réponses

  • Bonjour,

    Le terme général $\displaystyle u_n = {(-1)^n \over \ln(n) + \cos(n)}, n \in \N$ existe pour tout $\displaystyle n \in \N$, est réel, tend vers $0$ en valeur absolue (quand $n$ tend vers $+\infty$), et est décroissant en valeur absolue (au moins à partir d'un certain rang) et est alterné (au moins à partir d'un certain rang pour que le logarithme domine le cosinus changeant de signes) : donc la série $\displaystyle \sum_{n \geq 0} u_n$ converge. C'est le théorème du cours : critère de convergence des séries alternées.

    C'est faux : la fonction oscille...
  • bonsoir

    ta série alternée converge vers une limite comprise entre 1 et 2

    on prendra par commodité n = 3, 4, 5.....etc..(ce qui ne change pas la nature de la série)

    tu procèdes par encadrement : -1 < cosn < 1 (strictement)

    d'où l'encadrement du terme général $u_n$ pris en valeur absolue de la série :

    $\frac{1}{ln(n) + 1} < |u_n| < \frac{1}{ln(n) - 1}$ donc $|u_n| \sim \frac{1}{ln(n)}$

    or on sait que la série alternée de terme général $\frac{(-1)^n}{ln(n)}$ avec n = 2, 3........
    converge vers une limite positive

    la constante 1 ajoutée ou soustraite à ln(n) pour n > 2 strictement ne change rien à la nature de la série

    et donc ta série doublement alternée converge

    cordialement
  • YvesM écrivait:
    > est décroissant en valeur absolue (au moins à partir d'un certain rang)

    Mais un rang assez lointain quand même...63996
  • Deux posts avec des raisonnements qui ne tiennent pas la route ...
  • dans ce genre de situation , je cherche un équivalent et je le retranche de la suite
    La suite $w_n=u_n-\frac {(-1)^n}{\ln(n)}$ ne permet pas de conclure?
    edit Apres avoir avoir développé les calculs, ça ne marche pas
    Le 😄 Farceur


  • Ben, conclus alors !
  • Bonjour,

    On considère une somme finie pour additionner deux termes consécutifs : $\displaystyle S_N = \sum_{n=1}^{N} u_n = \sum_{n=1}^{N/2} (u_{2n } + u_{2n-1 } )$ avec $\displaystyle u_n = {(-1)^n \over \ln(n) + \cos(n)}$ : on ne pinaille pas sur le $\displaystyle N/2$...

    On a donc $\displaystyle S_N = \sum_{n=1}^{N/2} \Big( {1 \over \ln(2n) + \cos(2n) }- {1 \over \ln(2n-1) + \cos(2n-1)} \Big) =\sum_{n=1}^{N/2} {\ln(1-{1 \over 2n}) +\cos(2n-1)-\cos(2n) \over (\ln(2n) + \cos(2n) )(\ln(2n-1) + \cos(2n-1))} .$

    Le terme $\displaystyle {\ln(1-{1 \over 2n}) \over (\ln(2n) + \cos(2n) )(\ln(2n-1) + \cos(2n-1))} $ se comporte comme $\displaystyle {1 \over n \ln^2 n}, (n \to +\infty)$ et converge d'après Bertrand.

    Pour traiter le terme $\displaystyle \sum_{n=1}^{N/2} {\cos(2n-1)-\cos(2n) \over (\ln(2n) + \cos(2n) )(\ln(2n-1) + \cos(2n-1))} $ l'idée est de montrer qu'il vérifie ou pas le critère de Cauchy. Mais j'y travaille encore...
  • Bonjour,

    Pouvez-vous me dire si on peut conclure ?
    On sait, par les test de Dirichlet, que la série de terme general ${\sin(n) \over \ln(n)} $ converge ; que dire de la série de terme general $O({\sin(n) \over \ln(n)}) $ ?
  • Tu veux conclure par quel argument ?

    La série de terme général $\dfrac{(-1)^n}{n}$ converge par le même critère, pourtant la série de terme générale $0$ pour $n$ impair et $\dfrac{(-1)^n}{n}$ pour $n$ pair est $O\left(\dfrac{(-1)^n}{n}\right)$ mais ne converge pas.
  • @YvesM
    On ne peut rien dire ( sans convergence absolue)
    $\sum \frac {(-1)^n}n$ converge mais on ne sait rien sur $O(\frac {(-1)^n}n)$

    edit c'est un duplicata de ce qu' a dit skyfer :-P
    Le 😄 Farceur


  • Bonjour,

    Merci. Peut-on conclure ici ?
    On sait que la série de terme general ${\sin^2(n) \over \ln(n)}$ diverge par linearisation du numérateur comme somme d'une série convergente par le test de Dirichlet et d'une autre série divergente par Bertrand.

    Quid de $O({\sin^2(n) \over \ln(n)})$ ?
  • Je ne comprends toujours pas par quel argument tu souhaites conclure et encore moins ce que tu souhaites faire.

    ${\sin^2(n) \over \ln(n)} = O({\sin^2(n) \over \ln(n)})$ aboutit à une série divergente mais $0=O({\sin^2(n) \over \ln(n)})$ aboutit à une série convergente.
  • @Alexique
    Peux-tu faire suivre la question sur math.stackexchange aussi?
    Je crois que cette question va garder son secret jalousement et pour longtemps :-D
    Le 😄 Farceur


  • Of course, just here...
  • Une question :Est-ce que notre problème est il lié à une fonction dont les oscillations sont bornées en moyenne ?
  • voir plus bas
    Le 😄 Farceur


  • Puisque $n\mapsto \log(n)$ varie beaucoup plus lentement que $n\mapsto \cos(n)$, une idée serait de regrouper les termes par paquets de manière à profiter d'une moyennisation des cosinus. Plus précisément, si on pose :
    $$
    a_N = \sum_{n = N^2+1}^{(N+1)^2} \frac{(-1)^n}{\log(n) + \cos(n)},\qquad N \geq 1,
    $$
    on s'attend à avoir $a_N$ d'un ordre de grandeur proche de $\dfrac{1}{\log(N)}$, qui tend vers $0$ pour $N \to \infty$.
    On devrait même pouvoir trouver $(b_n)$ croissante de limite $+\infty$ et telle que les sommes partielles $N\mapsto \displaystyle \sum_{n=1}^N a_n\, b_n$ restent bornées, ce qui entrainerait la converge de la série de terme général $a_n$, et on devrait pouvoir en déduire le résultat voulu.

    Il reste à voir si ce programme est tractable techniquement...


    En guise d'échauffement, je propose de commencer par démontrer que pour tout réel $L$ assez grand,
    $$
    \sup_{x \geq 0}\; \left|\sum_{n \leq x} \frac{(-1)^n}{L + \cos(n)}\right| < \infty.
    $$
  • On pourrait dans la lignée de Siméon démontrer qu'on a avec $\varepsilon>0$:
    $$\sup_{t\in\mathbb{R}}|f(t+T)-f(t)| \leq \varepsilon$$
    Avec $f(x)=\frac{1}{ln(x+1)+cos(x+1)}$
    La fonction serait alors presque périodique (au sens de Bohr)
    Cordialement.
  • Bon, je vais déjà étudier le lien sur math.stackexchange.com ...
  • Je crois que la question était bien résolue ici https://math.stackexchange.com/questions/21175/convergence-of-the-series-sum-limits-n-2-infty-frac-1n-lnn puisque
    $\displaystyle \sum_{n=n_0}^\infty\frac{(-1)^n}{\ln n+\cos n}=
    \sum_{n=n_0}^\infty \frac{(-1)^n}{\ln(n)} \frac{1}{1+\frac {\cos n}{\ln(n)}}=
    \sum_{n=n_0}^\infty \sum_{k=0}^\infty \frac{(-1)^n}{\ln(n)} \frac{{(-1)^k}\cos^k(n)}{\ln^k(n)}$
    Le 😄 Farceur


  • Y'a a quand même un truc qui m'échappe. Peut-on trouver une suite $a_n$ dans $[-1,1]$ telle que la série $\sum \dfrac{(-1)^n}{\log(n) + a_n}$ diverge ? Dans le pire des cas on prend $a_n=-1$ pour $n$ pair et $a_n=1$ pour $n$ impair. Mais est-ce que $\sum \dfrac{(-1)^n}{(\log(n)+(-1)^{n+1})}$ diverge ?
  • oui skyffer ta série diverge comme somme de deux série de natures différentes
    Le 😄 Farceur


  • Peut-être peux-tu préciser de quelle somme il s'agit ?
  • Tu poses $v_n=$ta suite - $\frac {(-1)^n}{\ln(n)}$
    par magie $v_n$ est du signe constant et équivalente à $\frac 1{\ln^2(n)}$
    Le 😄 Farceur


  • Je ne crois pas à la magie ;-) Merci de ta réponse.
  • @skyffer3 : @gebrane0 veut dire que la réalité est magique ;-).
  • @gebrane0 : effectivement ! La solution de joriki correspond exactement à ce que j'ai fait pour mon échauffement. Mais je n'avais pas remarqué que ça résolvait directement le problème. La fatigue...
  • @Alexique
    J'attends avec impatience comment tu vas expliquer la preuve de MSE sur le Forum ilemaths
    Tu as du boulot sur la planche :-D
    Le 😄 Farceur


  • @gebrane0 : je n'en avais pas l'intention d'autant que ce n'est pas moi qui ait initié le topic sur ilemaths ! Je suis surtout satisfait de constater que certaines choses que je pensais ne pas être à ma portée... ne le seront définitivement jamais donc aucun regret !
  • Je propose, question liée, de montrer que $$\sum_{n\geq 1} \dfrac{\exp ( i nx)}{\ln(n)}$$ diverge pour tout $x$ réel et ceci par des transformations d'Abel.
  • EXERCICE
    @Satan : Test de Dirichlet

    Ta série converge pour (presque) tous les réels $x$.
  • Je pense répondre à la question initiale par ce biais : en sommant le sinus et montrant que les $1/log^2$ tendent vers zéro...Mais les log+cos ne sont pas décroissants...Désolé.
  • Je propose une fonction holomorphe : $$f(a)=\sum_{n\geq1} \dfrac{(-1)^n}{\ln(n)+ \cos (an)}$$ je crois que si la série, somme de fonctions holomorphes converge, alors elle est holomorphe et sa dérivée est somme des dérivées ; mais en dérivant la série en $a$, ça diverge... Donc ça doit diverger parfois en $a$.
  • @Satan
    Tu ne lis pas les messages précédents ! :-D
    Une fois sur la monotonie de ln+cos et une sur la nature de $\sum_{n\geq1} \dfrac{(-1)^n}{\ln(n)+ \cos (an)}$ qui diverge en $a=\pi$
    Le 😄 Farceur


  • Oui, mais ça diverge autour de toute valeur de $a$, pas qu'en $\pi$...
  • Satan écrivait :
    > Oui, mais ça diverge autour de toute valeur de $a$

    Peux-tu m'expliquer le sens de cette phrase ?
    Le 😄 Farceur


  • Pour tout nombre réel $a$ et tout voisinage $V_a$ de $a$, il existe $a'$ tel que la série diverge en $a'$.

    Je ne sais pas par ailleurs si l'on n'a pas un théorème d'analyse complexe qui dirait que si la série $S(z)$ de fonctions holomorphes converge sur $\C \setminus \R$ et si la série $S(z)$ converge en $x \in \R$, alors la série converge dans un voisinage de $x$?
  • Bonjour,

    D'abord merci à @Alexique pour cette série alternée coriace. Voici ma démonstration que cette série diverge.

    Soit $n$ un entier non nul, soit la suite numérique $u$ définie par $\displaystyle u_n = {(-1)^n \over \ln(n) + \cos(n)}, n \geq 1$, soit la série $\displaystyle U = \sum_{n \geq 1} u_n.$ On cherche la nature de la série $U.$

    Je rédige par proposition pour plus de rapidité.

    Soit $\displaystyle x \in \{-1, 1\}$ et la suite $d(x)$ définie par, pour tout $\displaystyle n \geq 1$, $\displaystyle 2 d_n(x) = {1 \over \ln^2(2n)(\ln(2n) + x)} - {1 \over \ln^2(2n+1)(\ln(2n+1) -x)} + {\cos(4n) \over \ln^2(2n) (\ln(2n) + x)} - {\cos(4n+2) \over \ln^2(2n+1) (\ln(2n+1) - x)}.$

    Proposition 0 : la suite $u$ est définie sur $\displaystyle n \geq 1$ et numérique.
    Proposition 1 : la série $U$ est alternée.
    Proposition 2 : $\displaystyle u_n \to 0, (n \to \infty).$
    Proposition 3 : la série $U$ est de même nature que la série $v$ définie par $\displaystyle v_n = u_{2n} + u_{2n+1}, n \geq 1.$
    Proposition 4 : $\displaystyle v_n - ({1 \over \ln(2n)} - {1 \over \ln(2n+1)}) + {\cos(2n) \over \ln^2(2n)} - {\cos(2n+1) \over \ln^2(2n+1)} = {\cos^2(2n) \over \ln^2(2n)(\ln(2n) + \cos(2n))} - {\cos^2(2n+1) \over \ln^2(2n+1)(\ln(2n+1) + \cos(2n+1))}.$
    Proposition 5 :
    $\displaystyle {\cos^2(2n) \over \ln^2(2n)(\ln(2n) +1)} - {\cos^2(2n+1) \over \ln^2(2n+1)(\ln(2n+1) -1)} \leq v_n - {\ln(1+{1 \over 2n}) \over \ln(2n)\ln(2n+1)} + {\cos(2n) \over \ln^2(2n)} - {\cos(2n+1) \over \ln^2(2n+1)} \leq \\ \displaystyle \leq {\cos^2(2n) \over \ln^2(2n)(\ln(2n) -1)} - {\cos^2(2n+1) \over \ln^2(2n+1)(\ln(2n+1) +1)}.$
    Proposition 6 : $\displaystyle {\cos^2(2n) \over \ln^2(2n)(\ln(2n) +x)} - {\cos^2(2n+1) \over \ln^2(2n+1)(\ln(2n+1) -x)} = d_n(x).$
    Proposition 7 : $\displaystyle d_n(+1) \leq v_n - {\ln(1+{1 \over 2n}) \over \ln(2n)\ln(2n+1)} + {\cos(2n) \over \ln^2(2n)} - {\cos(2n+1) \over \ln^2(2n+1)} \leq d_n(-1) .$
    Proposition 8 : $\displaystyle {1 \over \ln^2(2n)(\ln(2n) + x)} - {1 \over \ln^2(2n+1)(\ln(2n+1) -x)} = {-2 x \over \ln^2(2n) (\ln^2(2n)-1)} + O({1 \over n \ln^4(n)}).$
    Proposition 9 : $\displaystyle 0 \leq v_n - {\ln(1+{1 \over 2n} )\over \ln(2n)\ln(2n+1)} + {\cos(2n) \over \ln^2(2n)} - {\cos(2n+1) \over \ln^2(2n+1)} - d_n(+1) \leq d_n(-1)-d_n(1)$ et $\displaystyle d_n(-1)-d_n(1) = \frac12 {\cos(4n) \over \ln^2(2n) (\ln(2n) -1)} - \frac12 {\cos(4n+2) \over \ln^2(2n+1) (\ln(2n+1) +1)} -\frac12 {\cos(4n) \over \ln^2(2n) (\ln(2n) +1)} + \frac12 {\cos(4n+2) \over \ln^2(2n+1) (\ln(2n+1) -1)} + \\ \displaystyle + O({1 \over n \ln^4(n)}).$
    Erreur de calcul : la différence $d(-1) - d(1)$ n'annule pas le terme en $x$ de la proposition 8 mais le double !
    Proposition 10 : énoncer les résultats du cours sur les séries de Bertrand, énoncer le test de Dirichlet, et montrer que les séries suivantes convergent : $\displaystyle \sum_{n \leq 1} {\ln(1+{1 \over 2n} )\over \ln(2n)\ln(2n+1)} $, $\displaystyle \sum_{n \leq 1}{\cos(2n) \over \ln^2(2n)} $, $\displaystyle \sum_{n \leq 1} {\cos(2n+1) \over \ln^2(2n+1)} $, $\displaystyle \sum_{n \leq 1} {\cos(4n) \over \ln^2(2n) (\ln(2n) + x)}$, $\displaystyle \sum_{n \leq 1} {\cos(4n+2) \over \ln^2(2n+1) (\ln(2n+1) - x)}$, $\displaystyle \sum_{n \geq 2} O({1 \over n \ln^4(n)}).$
    Proposition 11 : montrer que cette série diverge $\displaystyle \sum_{n \leq 1} {-2 \over \ln^2(2n) (\ln^2(2n)-1)} .$
    Proposition 12 : soit une série $A$ et une série $B$ telles que, pour tout $\displaystyle n \geq 1$, $\displaystyle 0 \leq a_n \leq b_n$ et la série $B$ est convergente. Montrer que la série $A$ converge.
    Proposition 13 : soit une série $A$ et une série $B$ telles que, pour tout $\displaystyle n \geq 1$, $\displaystyle a_n + b_n = c_n$ avec la série $C$ convergente. Montrer que les séries $A$ et $B$ sont de même nature.

    Démonstration :
    Conclure que la série $U$ diverge.

    La série $\displaystyle v - d(+1)$ converge ; les séries $v$ et $\displaystyle d(+1)$ sont de même nature ; la série $\displaystyle d(+1)$ est la somme de séries qui convergent et d'une série divergente, donc elle diverge. La série $v$ diverge. La série $U$ diverge.
  • @YvesM
    joriki a démontré que cette série converge, problème !
    (Je n'ai pas regardé tes calculs pour le moment)
    Le 😄 Farceur


  • Bonjour,

    Je travaille seul... mais j'ai fait une erreur (voir correction). Donc je n'arrive toujours pas à m'en sortir. Je cherche encore un peu et je regarderai l'autre démonstration de 'joriki'. Avec un développement limité, sans ne marche pas, à n'importe quel ordre ; avec le test de Cauchy, je n'y arrive pas car le terme $\sin(2n + \frac12)$ change de signe trop souvent ; avec des majorations (ma tentative plus haut), il reste toujours un terme qui gâche la fête... ; avec une comparaison avec une intégrale, même histoire où la majoration diverge...
  • La preuve de joriki n'est pas claire, il y a ce point je ne vois pas Mk can be bounded by a power of k,
    @Simeon@...
    Stp et si tu as un peu de temps
    peux-tu nous expliquer la preuve de joriki
    Le 😄 Farceur


  • Bonjour !
    Merci @gebrane0 de poser cette question : je séchais aussi sur cette majoration de $M_k$ par une puissance de $k$.

    Consolation de "minable" de voir que je ne suis pas le seul à caler !
  • Soit $j$ un entier et soit $x$ tel que $j + \pi \in x + 2\pi\mathbb Z$ et $|x| < \pi$. On a alors :
    $$1 - \cos(j+\pi) = 1 - \cos(x) = 2\sin\left(\frac x2\right)^2 \geq \frac{2|x|^2}{\pi^2}.$$
    Mais par ailleurs, $|x|$ ne peut pas être trop petit ! Effet, il existe $\alpha > 0$ tel que $|\pi - \frac pq| \geq |q|^{-(\alpha+1)}$ pour tout $(p,q)\in\Z\times\Z^*$ avec $|q|$ assez grand. Si on note $q$ l'entier impair tel que $x = j - q\pi$, on obtient alors
    $$
    |x| =\left|\pi - \frac{j}{q}\right| \times |q| \geq |q|^{-\alpha}
    $$
    pour tout $j$ assez grand car $q\pi = j + O(1)$. Ainsi $\dfrac{2}{1- \cos(j+\pi)} = O(j^{2\alpha})$, d'où finalement :
    $$
    \boxed{\displaystyle\sum_{j=0}^k \frac{2}{1-\cos(j+\pi)} = O(k^{2\alpha+1})}
    $$
  • Merci Simeon pour cette rapidité et cette efficacité. Ce n’étais pas si évident! encore merci
    Le 😄 Farceur


  • Je t'en prie, surtout que j'ai admis l'existence de $\alpha$, ce qui est la partie la plus difficile. Pour ceux que ceci intéresse, l'article de Maurice Mignotte semble assez clair : http://archive.numdam.org/article/MSMF_1974__37__121_0.pdf (en français)

    P.S. les spécialistes connaissent maintenant de meilleures bornes.
  • Siméon écrivait $$2\sin\left(\frac x2\right)^2 \geq \frac1\pi |x|^2.$$
    Si $x$ est proche de $\pi$, bof... mais bon on s'en fiche, ça ne change pas les $O$ et le reste. C'était histoire d'intervenir dans ce topic...
    Mais j'aurais pu m'en passer...
  • Alexique écrivait :
    > C'était histoire d'intervenir dans ce topic... Mais j'aurais pu m'en passer...

    Une idée, pourquoi ne pas reprendre la demo de joriki ici et la rendre plus digérable pour les visiteurs de 2030
    (on ne sera pas là peut-être pour les guider !)
    Le 😄 Farceur


  • En effet, les constantes sont délirantes. Je corrige !
  • Il y a une preuve rédigée proprement dans la rms du résultat de ce topic ^^

    https://www.rms-math.com/index.php?option=com_staticxt&Itemid=65&staticfile=RMS117-497.html
Connectez-vous ou Inscrivez-vous pour répondre.